New homeowners hire a painter to paint rooms in their house. The painter pays $100 for supplies and charges the homeowners $25 for each room they want painted.

Which of the following graphs shows the relationship between the amount of money the painter earns, in dollars, and the number of rooms he paints?

a coordinate grid with the x axis labeled rooms painted and the y axis labeled amount of money earned and a line going from the point 0 comma 100 through the point 4 comma 0
a coordinate grid with the x axis labeled rooms painted and the y axis labeled amount of money earned and a line going from the point 0 comma negative 100 through the point 4 comma 0
a coordinate grid with the x axis labeled rooms painted and the y axis labeled amount of money earned and a line going from the point 0 comma 100 through the point 4 comma 200
a coordinate grid with the x axis labeled rooms painted and the y axis labeled amount of money earned and a line going from the point 0 comma negative 100 through the point 4 comma negative 200

Answers

Answer 1

The correct graph is: a coordinate grid with the x-axis labeled rooms painted and the y-axis labeled amount of money earned and a line going from the point (0,100) through the point (4,200).

What is a Graph?

In math, a graph can be defined as a pictorial representation or a diagram that represents data or values in an organized manner.  

The points on the graph often represent the relationship between two or more things.

The painter pays $100 for supplies, which is a fixed cost, and charges $25 for each room painted, which is a variable cost.

Therefore, the relationship between the amount of money the painter earns and the number of rooms he paints can be represented by a linear equation in slope-intercept form:

Amount earned = (price per room) x (number of rooms painted) + (fixed cost)

In this case, the price per room is $25, the number of rooms painted is the x-value, and the fixed cost is $100. Thus, the equation is:

Amount earned = 25x + 100

To graph this equation, we can plot two points and draw a line through them. One point is when no rooms are painted, which gives the painter $100 for the fixed cost:

(0,100)

The other point is when four rooms are painted, which gives the painter:

Amount earned = 25(4) + 100 = 200

So the other point is:

(4, 200)

The correct option :

a coordinate grid with the x axis labeled rooms painted and the y axis labeled amount of money earned and a line going from the point 0 comma 100 through the point 4 comma 200.

Learn more about Graph at:

https://brainly.com/question/17267403

#SPJ1


Related Questions

Tomas used 3 [tex]\frac{1}{2}[/tex] of flour and now has 1 and two-thirds cups left. Which equation can he use to find f, the number of cups of flour he had to begin with?

f + 3 and one-third = 1 and two-thirds
f minus 3 and one-third = 1 and two-thirds
3 and one-third f = 1 and two-thirds
f divided by 3 and one-third = 1 and two-thirds

Answers

Answer:

B

Step-by-step explanation:

Answer:

I think It's b or a

Step-by-step explanation:

did this like 4 weeks ago.

What function is represented in the table?
Select one:
y = 3(2^x)
y = 2(3^x)
y = 2(.5^x)
y = 3^x

Answers

The exponential function represented in the table is given as follows:

y = 3^x.

How to define an exponential function?

An exponential function has the definition presented as follows:

y = ab^x.

In which the parameters are given as follows:

a is the value of y when x = 0.b is the rate of change.

From the table, we have that when x=  0, y = 1, hence the parameter a is given as follows:

a = 1.

When x is increased by one, y is multiplied by three, hence the parameter b is given as follows:

b = 3.

Hence the function is:

y = 3^x.

More can be learned about exponential functions at brainly.com/question/2456547

#SPJ1

PLEASE HELP ( WILL GIVE BRAINLIEST)

Answers

The amount of money Laurie would pay to have the pond cleaned is equal to $3,161.85.

How to calculate the volume of a hemisphere?

In Mathematics and Geometry, the volume of a hemisphere can be calculated by using the following mathematical equation (formula):

Volume of a hemisphere = 2/3 × πr³

Where:

r represents the radius.

Note: Radius = diameter/2 = 13/2 = 6.5 feet.

By substituting the given parameters into the formula for the volume of a hemisphere, we have the following;

Volume of a hemisphere = 2/3 × 3.14 × (6.5)³

Volume of a hemisphere = 574.881 ft³

Cost = 5.50 × 574.881 ft³

Cost = $3,161.85

Read more on volume of a sphere here: brainly.com/question/20394302

#SPJ1

A company has recently been hiring new employees. Today the company has 33% more employees than it did a year ago. If there are currently
66,500 employees, how many employees did the company have a year ago?
employees

Answers

If the company has 33% more employees than it did a year ago and there are currently 66,500 employees, the number of employees the company had a year ago was proportionately, 50,000 employees.

What is proportion?

Proportion refers to the ratio of one quantity or value compared to another.

Proportions are fractional values, representing using fractions, decimals, or percentages.

The current population of a company's employees = 66,500

The percentage increase in the population from last year's = 33%

Proportionately, last year's population = 100%, while this year's population is 133% (100 + 33).

If 66,500 = 133% (1.33), then 100% = 50,000 (66,500 ÷ 1.33)

Learn more about proportions at https://brainly.com/question/1496357.

#SPJ1

Male 16 15 2 33
Female 14 11 18 43
Total 30 26 20 76
If one student is chosen at random,

Find the probability that the student was female:

Find the probability that the student was female AND got a "C":

Find the probability that the student was female OR got a "C":

If one student is chosen at random, find the probability that the student was female GIVEN they got a 'C':

Answers

a) The probability that the student was female is 43/76

b) The probability that the student was female AND got a "C" is 9/38

c) The probability that the student was female OR got a "C" is 45/76

We have,

             A   B   C  

Male     16  15  2  33

Female 14  11  18  43

Total     30 26 20 76

a) The probability that the student was female

= 43/76

b) The probability that the student was female AND got a "C"

= 18/76

= 9/38

c) The probability that the student was female OR got a "C"

= 43 /76 + 20/76 - 18/76

= 45/76

Learn more about Probability here:

https://brainly.com/question/30034780

#SPJ1

Help please!! I need help!!! I will mark you brainlest!​

Answers

Answer:

2(4.5) + (1/2)(4)(2) = 9 + 4 = 13 square feet

13 square feet it is

A circular arc of length 9 feet subtends a central angle of 65 degrees. Find the radius
of the circle in feet.

Answers

The radius of the circle is approximately 7.9 feet.

What is the radius of the circle?

The arc length formula (if θ is in degrees) is expressed as:

s = 2πr × (θ/360°)

Where r is radius and θ is the central angle subtended by the arc.

Given that:

Arc length s = 9ft

Central angle subtended by the arc = 65°

Radius r = ?

Plug the given values into the above formula.

s = 2πr × (θ/360°)

9ft = 2 × 3.14 × r × (65/360°)

r = 9ft / 1.133888

r = 7.9 ft

Therefore, the radius is 7.9 ft.

Learn more about arc of length here: https://brainly.com/question/16403495

#SPJ1

1. Out of 318 seventh and eighth grade CAVA students there were 48 students that chose Music as their elective
course and the rest chose World Language. There were a total of 124 eighth grade students and 108 of them
chose World Language as their elective.
Use this information to complete the two-way table completely.
Enter your answer by filling in the boxes to complete the table (5 pts)
Answer:
8th Grade
7th Grade
Totals
Music
48
World Language
108
Totals
124
318

Answers

The students in 8th class who choose music are 16 in number,

The students in 7th class who choose music are 32 in number

The total who choose world language is 270

The totals who are in 7th grade be 194

The number of students who choose world language in 7th class are 162

Let x be the students in 8th class who choose music

x+108=124

x=124-108

x=16 students

Now  y be the students in 7th class who choose music

16+y=48

y=48-16

y=32

Let z be the total who choose world language

48+z=318

z=318-48

z=270

Now the totals who are in 7th grade be A

124+A=318

A=318-124

A=194

B be the number of students who choose world language in 7th class

Now 32+B=194

B=194-32

B=162

To learn more on Equation:

https://brainly.com/question/10413253

#SPJ1

Which function is a translation one unit right of the function f(x) = log x?

h(x) = log x + 1
g(x) = log x-1
j(x) = log(x + 1)
k(x) = log(x - 1)

Answers

Answer:

The function that is a translation one unit right of the function f(x) = log x is k(x) = log(x - 1).

When we shift a function f(x) one unit right, we replace x with (x - a) where "a" is the amount of the shift. In this case, "a" is equal to 1. Therefore, the function becomes:

f(x - 1) = log(x - 1)

This means that the function k(x) = log(x - 1) is obtained by shifting the function f(x) one unit to the right.

A 138 foot antenna is on top of the top of a tall building. From a point on the ground, the angle of elevation to the top of the antenna is 30.5゚ while the angle of elevation to the bottom of of the antenna from the same point is 23.5゚How tall is the building​

Answers

Answer:

289

Step-by-step explanation:

Let's call the height of the building "h" and the distance from the point on the ground to the base of the building "d".

From the point on the ground, the distance to the top of the antenna is d + 138, and the distance to the bottom of the antenna is d.

Using trigonometry, we can set up two equations:

tan(30.5°) = (d + 138) / h (1)

tan(23.5°) = d / h (2)

We can solve equation (2) for d:

d = h tan(23.5°)

Substituting this into equation (1), we get:

tan(30.5°) = (h tan(23.5°) + 138) / h

Simplifying:

tan(30.5°) = tan(23.5°) + 138/h

tan(30.5°) - tan(23.5°) = 138/h

h = 138 / (tan(30.5°) - tan(23.5°)) ≈ 289.12 feet

Therefore, the height of the building is approximately 289.12 feet.

PLEASE HELP ILL DO SNYTHING PLSPLSPLS

5. Beth is 63 years old. She loves to bike. She decides to look at other clubs to join. Why do you think she didn’t want to join this club? Explain your answer using the stem-and-leaf plot above. (1 point)

Answers

Beth could not join the club because the highest age which is allowed here is only 55 years

Since, A statistical expression obtained from a list of data that refers an abnormal gap from other values.

The statistical rules that instruct us to divides the data or observation values into four parts.

After analyzing the stem leaf diagram, we noticed the youngest age allowed to join the club is 10 years and the club allow highest 55 years old to join.

the plot also defines that the number of members is declined with the age. There is only person in the club who is 55 years. There are few members with the age over 40 years.

hence, Beth did not join the club because her age was above the highest age that allowed in this club.

to know more about Stem Leaf diagram visit:

brainly.com/question/29479023

#SPJ1

Find the missing angle

Answers

Answer:

I Think 62°

Step-by-step explanation:

I think it’s the answer is 62

Michael checked several websites and stores around town for the television he wanted to purchase. He saw eight different prices: $273 $267 $276 $297 $264 $294 $264 $269 What is the mean, median, and mode of this data set?

Answers

The mean is 270.75 and the median is 271. While there is no single mode, then we conclude that our mode is 264 and 294 respectively.

Understanding how to calculate mean, median, mode

By definition:

Mean is the sum of all the numbers in the set divided by the total number of numbers.

Median is the middle value in the set when the numbers are arranged in order.

Mode is the number that appears most frequently in the set.

If we re-arrange the data given in ascending order, we have:

264 264 267 269 273 276 294 297

To calculate Mean:

Add up all the numbers and divide by the total number of numbers:

Mean = (264 + 264 + 267 + 269 + 273 + 276 + 294 + 297) / 8

Mean = 270.75

To calculate Median:

We need to find the middle number in the set. Since there are 8 numbers in the set, the median is the average of the 4th and 5th numbers:

Median = (269 + 273) / 2

Median = 271

To calculate Mode:

We need to determine which number appears most frequently in the set. In this case, there are two numbers that appear twice (264 and 294), and the rest of the numbers appear only once. Therefore, there is no single mode for this data set.

Mode = 264 and 294

Learn more about median here:

https://brainly.com/question/26177250

#SPJ1

Question 2
Plot the coordinates on the number line provided and then find the coordinate of the indicated point
on a number line that partitions the segment into the given ratio.
(The numbers are not aligned properly to the marks, so redraw the number line.)
A is at -2 and B is at 14. Find the point, T, so that T is three-fourths of the distance from A to B.
-10
5
10

Answers

Note that  in the line graph presented, 3/4 the distance from A to B is 12, thus, t = 12.

what is the explanation about this?

Given:

A (- 2 , 0) and B(14, 0)

Using distance formula,      we can determine the distance between coordinates given:

d    = √ ( (x2 - x1)² + ( y2 - y1) ²)

In this case,  x1 = -2  y1 = 0,

x2 = 14, and y2 =  0.

Substituting these     values into the formula, we get:

d = √(  (14 -   (-2 ) ) ² + (0 - 0) ²)

= √((16)² + (0)² )

= √(256)

= 16

The distanc between the points (-2,0) and (14,0) is 16   units.

Recall that we need to find the position of t which    is 3/4 the distance between A and B.

That is:

3/4 x 16

t = 12.


Learn more about line graphs:
https://brainly.com/question/13298277
#SPJ1

Hurry up please time limit

State the domain and range and tell if the graph is a function yes or no?
What’s the domain and range ?

Answers

Answer:No, X=-3, yly =-4,0,4

Step-by-step explanation:

Why is it essential to maximize the volume for a fixed surface area for cost and material waste?

Answers

It is essential to maximize the volume for a fixed surface area for cost and material waste.

Because it can help to optimize the efficiency of material usage and minimize the cost of production.

In many manufacturing and construction processes, the cost of materials is a significant expense.

By maximizing the volume of an object for a fixed surface area, we can reduce the amount of material needed to construct the object.

This means that we can save money on material costs and reduce waste by using fewer materials overall.

In addition, minimizing the surface area of an object for a fixed volume can also help to reduce production costs.

This is because the surface area of an object is often a factor in determining the time and labor required to produce it.

By minimizing the surface area, we can reduce the amount of time and labor needed to produce the object.

Which can help to reduce production costs.

Therefore, maximizing volume for fixed surface area is an essential consideration in many manufacturing and construction processes.

It help to optimize material usage, minimize waste, and reduce production costs.

learn more about volume here

brainly.com/question/15545998

#SPJ1

In this circle, mQR = 72°.
What is m/QPR?
A. 18°
B. 24°
C 36°
D. 72°

Please show work or give an explanation please

Answers

Answer:

< QPR = 36

Step-by-step explanation:

Inscribed Angle = 1/2 Intercepted Arc

< QPR = 1/2 ( 72)

< QPR = 36

Select the correct answer. What is the solution to 4|x − 3| + 1 = 1? A. x = 3 B. x = 3 or x = 4 C. x = 3 or x = 6 D. No solutions exist.

Answers

Answer:

a  

Step-by-step explanation:

We can start solving the given equation by isolating the absolute value term:

4|x - 3| = 0

Since the absolute value of any number is always non-negative, the only way for the left-hand side of the equation to be zero is if the absolute value term is zero. This means:

|x - 3| = 0

And the only solution to this equation is:

x - 3 = 0

x = 3

Therefore, the correct answer is A. x = 3.

Find the values of x and y

Answers

Answer:

x = 8.4

y = 23.6

Step-by-step explanation:

tan31 = x/14

x = tan31(14) = 8.412

y² = (8.412)² + (22)² = 554.7626

y = √554.7626 = 23.55

Find the volume of rectangular prism. (You should have at least 2 steps shows in your work!)(show all decimal places or a fraction)

Answers

The volume of this rectangular prism include the following: 84.375 cubic meters.

How to calculate the volume of a rectangular prism?

In Mathematics and Geometry, the volume of a rectangular prism can be calculated by using the following formula:

Volume of a rectangular prism = L × W × H

Where:

L represents the length of a rectangular prism.W represents the width of a rectangular prism.H represents the height of a rectangular prism.

By substituting the given dimensions (parameters) into the formula for the volume of a rectangular prism, we have the following;

Volume of rectangular prism = 5 × 4 1/2 ×  3 3/4

Volume of rectangular prism = 5 × 4.5 ×  3.75

Volume of rectangular prism = 84.375 cubic meters.

Read more on volume of prism here: brainly.com/question/21012007

#SPJ1

Which number line shows a graph of the inequality x < 6

Answers

The number line that shows the inequality x < 6 is given by the following option:

Option D.

What are the inequality symbols?

The four inequality symbols, along with their meaning on the number line and the coordinate plane, are presented as follows:

> x: the amount is greater than x -> the number is to the right of x with an open dot at the number line. -> points above the dashed horizontal line y = x on the coordinate plane.< x: the amount is less than x. -> the number is to the left of x with an open dot at the number line. -> points below the dashed horizontal line y = x on the coordinate plane.≥ x: the amount is at least x. -> the number is to the right of x with a closed dot at the number line. -> points above the solid vertical line y = x on the coordinate plane.≤ the amount is at most x. -> the number is to the left of x with a closed dot at the number line. -> points above the dashed vertical line y = x on the coordinate plane.

The inequality for this problem is given as follows:

x < 6.

Which is composed by the numbers to the left of x = 6, with an open circle, due to the open interval.

More can be learned about inequalities at brainly.com/question/25275758

#SPJ1

What is 3/10 + 1/4 + 4/5 enter your answer in the box as a mixed number in simplest form

Answers

The LCD is 20 so your answer should be
(27/20)or (1 and 7/20)

Answer:

[tex]1 \frac{7}{20} [/tex]

Step-by-step explanation:

Well you should find a common denominator for 10, 4 and 5 first to be able to mix them. Let's take 20 because it's the smallest number we can convert 10, and 4 and 5 to.

So

[tex] \frac{3 \times 2}{10 \times 2} + \frac{1 \times 5}{4 \times 5} + \frac{4 \times 4}{5 \times 4} [/tex]

And you get

6/20 + 5/20 + 16/20 (now you can add them because they all have 20 as denominator) so = 27/20 = 1 7/20

The data set below gives the number of confirmed cases of malaria in Ethiopia.
Find the exponential model that best fits the data set and use the equation to estimate the number of malaria cases in 2017.



Year 2001 2002 2005 2007 2009 2010 2012

Malaria 392 428 539 451 1036 1158 1693

cases

In thousands

Answers

The number of malaria cases in 2017 is 2700.

Let the dependent variable, y is malaria cases in thousands and the independent variable, x is the year.

Here, x is the number of years starting in the year 2000.

The required model is obtained as [tex]y = 299e^{0.1295x}[/tex]

Now, the predicted number of malaria cases in the year 2017, that is, x = 17 years can be computed as,

[tex]y = 299e^{0.1295(17)}\\\\= 299(9.020\\\\= 2700[/tex]

Hence, the number of malaria cases in 2017 is 2700.

Learn more about exponential model click;

https://brainly.com/question/28596571

#SPJ1

Do the following using Matlab: Generate 10 numbers between 1 and 6, calculate the mean. Generate 100 numbers between 1 and 6. calculate the mean. Generate 1000 numbers between 1 and 6, calculate the mean. There is a command in Matlab to generate numbers. Do not include the numbers when you submit the project. The project should show the commands used and the results only The idea is to roll a die using Matlab commands, instead of doing it manually. Roll a die 10 times and record the outcome and then find the average of the outcomes. Then compare the means for the cases of 10, 100 & 1000. On page 2 of lecture 10, we found that the mean of rolling a die to be 3.5. Therefore, your answer will be close to 3.5 every time you repeat the experiment for larger numbers.

Answers

We can see that the means we got from Matlab are close to 3.5 for larger numbers of rolls.

We have,

To generate numbers between 1 and 6 in Matlab, we can use the "randi" command.

To generate 10 numbers, we can use:
```matlab
numbers10 = randi([1, 6], [1, 10]);
mean10 = mean(numbers10);
disp(mean10);
```

To generate 100 numbers, we can use:
```matlab
numbers100 = randi([1, 6], [1, 100]);
mean100 = mean(numbers100);
disp(mean100);
```

To generate 1000 numbers, we can use:
```matlab
numbers1000 = randi([1, 6], [1, 1000]);
mean1000 = mean(numbers1000);
disp(mean1000);
```

After running these commands, we will get the means for generating 10, 100, and 1000 numbers between 1 and 6.

As mentioned in the question, the mean of rolling a die is 3.5.

Therefore,

We can see that the means we got from Matlab are close to 3.5 for larger numbers of rolls.

Learn more about math lab here:

https://brainly.com/question/10772022

#SPJ11

if i have 7 shirts and 2 pants how many outfits can i make​

Answers

Answer:14

Step-by-step explanation:

2 pants can only go with the different shirts once. So it should be 14

Which compares the end behavior of the functions f and g?
f(x) = −17x − 9 g(x) = − 78
7
8
x + 20
A. For function f, as x → ∞
x




, f(x) → ∞
f
(
x
)




. Likewise, for function g, as x → ∞
x




, g(x) → ∞
g
(
x
)




.
B. For function f, as x → ∞

x




, f(x) → −∞
f
(
x
)





. Likewise, for function g, as x → ∞
x




, g(x) → −∞
g
(
x
)





.
C. For function f, as x → ∞
x




, f(x) → −∞
f
(
x
)



-

. However, for function g, as x → ∞
x




, g(x) → ∞
g
(
x
)




.
D. For function f, as x → ∞

x




, f(x) → ∞

f
(
x
)




. However, for function g, as x → ∞
x




, g(x) → −∞
g
(
x
)





.

Answers

Answer:

The correct option that compares the end behavior of the functions f and g is D.

For function f, as x → ∞, f(x) → -∞, which means that the function approaches negative infinity as x approaches infinity. This is because the leading term of the function is -17x, which approaches negative infinity as x approaches infinity.

For function g, as x → ∞, g(x) → -∞, which means that the function also approaches negative infinity as x approaches infinity. This is because the leading term of the function is -78/87x, which approaches negative infinity as x approaches infinity.

Therefore, both functions have the same end behavior, which is approaching negative infinity as x approaches infinity.


A rectangular fish tank 35 cm long, 30 cm wide and 25 cm high is filled with water to a depth of 20 cm.
Find the volume of water in the fish tank.

Give your answer in liters.

Answers

Answer:

First, we need to calculate the volume of the rectangular fish tank. The volume of a rectangular solid is given by the formula:

volume = length x width x height

Plugging in the values we have, we get:

volume = 35 cm x 30 cm x 25 cm = 26,250 cubic cm

Next, we need to find the volume of water in the fish tank. The volume of water is equal to the volume of the space the water occupies, which is a rectangular solid with length 35 cm, width 30 cm, and height 20 cm. The volume of this rectangular solid is:

volume = length x width x height = 35 cm x 30 cm x 20 cm = 21,000 cubic cm

To convert cubic centimeters (cm^3) to liters (L), we need to divide by 1000:

volume = 21,000 cm^3 ÷ 1000 = 21 L

Therefore, the volume of water in the fish tank is 21 liters.

Step-by-step explanation:

72+z rewrite the expression as a produc 72

Answers

Rewriting the expression based on the information will be 72(1 + z/72).

How to explain the expression

In order to transform 72 + z into a product based on 72, we can apply the distributive property of multiplication over addition. Essentially, this property maintains that:

a x (b + c) = a x b + a x c

By extension, we can create the following equation:

72 + z = 72 + 1 x z

In order to produce an equivalent expression as a multiple of 72 and another factor, we would then need to use the distributive rule once again:

72 + 1 x z = 72 x 1 + 72 x z/72

Simplification leads us to arrive at:

72 + z = 72(1 + z/72)

Our end result is expressed as a synthesis between 72 and the expression (1 + z/72), namely:

72(1 + z/72).

Learn more about expressions on

https://brainly.com/question/1859113

#SPJ1

PLEASE HELP (WILL GIVE BRAINLIEST

Answers

The exact volume of the soup can is 18.375 π cubic inches. So, correct option is C.

The volume of a cylinder can be found by multiplying the area of the base (circle) by the height of the cylinder. In this case, the diameter of the base is 3.5 inches, which means the radius is 1.75 inches (since radius = diameter/2).

So the area of the base can be calculated as:

π × (1.75 in)² = π × 3.0625 in²

The height of the cylinder is given as 6 inches.

Therefore, the exact volume of the soup can would be:

π × (1.75 in)² × 6 in = π × 3.0625 in² × 6 in = 18.375 π cubic inches (rounded to two decimal places)

So, correct option is C.

To learn more about volume click on,

https://brainly.com/question/9774275

#SPJ1

PLEASEEEE HELP MEMEMEMEM PLESSEEE I WILL GIVE U BRAINLIST QUICK

Answers

The numeric value of the function graphed is given as follows:

f(-8) = 6.

How to obtain the numeric value of the function?

The expression for the numeric value of the function in this problem is given as follows:

f(-8).

This means that the input is given as follows:

x = -8.

Passing a vertical line through x = -8, the value of y in which the vertical line crosses the graph is given as follows:

y = 6.

Hence the numeric value is given as follows:

f(-8) = 6.

Learn more about the numeric values of a function at brainly.com/question/28367050

#SPJ1

Other Questions
Tesar Chemicals is considering Projects S and L, whose cash flows are shown below. These projects are mutually exclusive, equally risky, and not repeatable. The CEO believes the IRR is the best selection criterion, while the CFO advocates the NPV. If the decision is made by choosing the project with the higher IRR rather than the one with the higher NPV, how much, if any, value will be forgone, i.e., what's the chosen NPV versus the maximum possible NPV? Note that (1) "true value" is measured by NPV, and (2) under some conditions the choice of IRR vs. NPV will have no effect on the value gained or lost. WACC: 6.75% 0 1 2 3 4 CFS -$1,100 $550 $600 $100 $100 CFL -$2,700 $650 $725 $800 $1,400 Instead of filling vacant instructor positions with new full-time faculty, Hope College can take advantage ofA) temporary employees.B) technological innovation.C) turnover reductions.D) new external hires.E) retraining. 46. A decreasing average collection period could be associated with: A. increasing sales.B. decreasing sales.C. increasing accounts receivable.D. increasing profits. A textile production facility produces curtains which are sold in home design stores in their area. Their gross sales in hundreds of dollars, S, is dependent on the number of curtains they produce, x, and can be modeled by the functionS(x)=20+2.5x.Draw the graph of the gross sales function by plotting its S-intercept and another point. Which drug can cause colchicine levels to go too high? Clarithromycin Levetiracetam Penicillin Terbinafine During the Trump administration, Secretary of Education Betsy DeVos came into the position with strong support for which of the following?charter schoolsvouchersincreased funding for public educationstrengthening teacher unions airline accessories has the following current assets: cash, $109 million; receivables, $101 million; inventory, $189 million; and other current assets, $25 million. airline accessories has the following liabilities: accounts payable, $112 million; current portion of long-term debt, $42 million; and long-term debt, $30 million. based on these amounts, calculate the current ratio and the acid-test ratio for airline accessories. (enter your answers in millions, not in dollars. for example, $5,500,000 should be entered as 5.5.) Find the value of x, y, and z. The work I did in the problem is how I got it wrong. Nurses must be aware of their own cultural values and beliefs to avoid biases when providing care to clients.True or false 2. Scientific management is based on economic incentives and the premise that there is "one best way" to perform any job. Find the measure of minor arc BC (labeled as x) These countries were part of NATO * Canada and Soviet Union Italy and United States United States and Soviets Belgium and Poland Helpppppppppppppppppp describe the theoretical basis of the humanistic approach to therapy. list the four assumptions on which phenomenological therapists operate. Which expressions are equivalent to 1/3 - 2/5 The standard deviation of the sample mean is equal to 1) The population variance 2) The sample variance 3) The sample error O 4) Question 10 (1 point) A local retail company found a sample mean of $1,000 using a sample size of 100. What is the 90% confidence level for the estimated mean if the estimated standard deviation of $20? Assume the t value is 1.65. 1) $996.7 to $1,003.3 O2) $992.5 to $1,008.5 3) $1,000.0 to $1,006.6 4) $996.0 to $1,004.0 suppose the federal reserve sets the reserve requirement at 14%, banks hold no excess reserves, and no additional currency is held. instructions: in part a, round your answer to one decimal place. in parts b and c, enter your answers as a whole number. if you are entering a negative number include a minus sign. a. what is the money multiplier? 7.1 b. how much will the total money supply change by if the federal reserve changes reserves by -$70 million? $ -500 million suppose the federal reserve wants to decrease the total money supply by $400 million. c. how much should the federal reserve change reserves to achieve this goal? $ million generally, when preferences for a good rise, demand for the good rises. if a perfectly competitive market starts in long-run equilibrium, holding all else constant, this will result in a higher market price, which will lead to in the industry and the market. this causes price to . economic losses; causes some firms to leave; rise further economic losses; attracts new firms into; fall positive economic profits; causes some firms to leave; rise further positive economic profits; attracts new firms into; fall What is this super hard equation in my 2nd grade state test : 1 + 1 x 1 / 1? The German economist and philosopher stressed that history is a continuous clash between conflicting ideas and forces. He believed that class conflict is necessary in order to produce social change and a better society.a. Max Weberb. Emilie Durkheimc. Karl Maxd. Frederick Schmidt